Anuncio

Colapsar
No hay ningún anuncio todavía.

Teorema Stokes

Colapsar
X
 
  • Filtro
  • Hora
  • Mostrar
Borrar todo
nuevos mensajes

  • 1r ciclo Teorema Stokes

    Hola debo hacer el siguiente problema pero no soy capaz:
    Verificar el teorema de Stokes para el triangulo de vertices (3,0,0) (0,3,0) (0,0,6)
    Yo empiezo por sacar la ecuacion del plano que lo contiene y me sale:
    2x+2y+z=6, a partir de hay debo sacar una parametrizacion del plano para sacar el dS pero no estoy seguro de si lo hago bién prove haciendo:
    x(u,v)=(u,v,6-2u-2v)
    Hago el rotacional de mi campo vectorial que me da: (-1,-1,2y)
    y por lo tanto me queda la siguientes integrales:
    -dydz-dxdz-2y*dydx
    Y ahora es donde tengo la mayor duda es en cuales serian los limites de integracion, a mi se me ocurrio poner por ejemplo para la primera integral
    y varia de 0 a tres y z varia con respecto a la recta contenida en el plano YZ, es decir varia de 6-2y a 0 y asi con todos los limites, pero me da que eso no va a estar bién
    Muchas Gracias

  • #2
    Re: Teorema Stokes

    El diferencial de superficie se encuentra con lo que se llama "producto vectorial fundamental de la superfície",


    Ahora bien, fíjate que tal y como has elegido la parametrización y = v. Así que la integral que te quedará será


    El dominio de integración en el plano (u, v), que no es más que (x, y), es un triángulo con vértices en (0, 0), (3, 0) y (0, 3). Sólo hay que quitar el valor de z de los límites (porque ya está parametrizado en función de u y v).

    Si elegimos integrar primero la v, por ejemplo, irá de 0 hasta 3-u. Luego, v irá de 0 a 3.
    La única alternativo a ser Físico era ser etéreo.
    @lwdFisica

    Comentario

    Contenido relacionado

    Colapsar

    Trabajando...
    X